Mathcenter Forum

Mathcenter Forum (https://www.mathcenter.net/forum/index.php)
-   อสมการ (https://www.mathcenter.net/forum/forumdisplay.php?f=18)
-   -   ขอโจทย์ A.M.-G.M.-H.M.หน่อยครับ (https://www.mathcenter.net/forum/showthread.php?t=3357)

CmKaN 12 ตุลาคม 2007 17:52

ขอโจทย์ A.M.-G.M.-H.M.หน่อยครับ
 
ก็ตามหัวข้อ ขอโจทย์ที่ใช้ A.M.-G.M.-H.M. หน่อยครับ พอดียังไม่ชำนาญ และค่อนข้างมั่วๆ ก็ขอแบบ ง่ายๆไปยากน่ะครับ:) ขอบคุณครับ:please:

หยินหยาง 12 ตุลาคม 2007 19:50

ไม่รู้จะง่ายไปหรือเปล่านะ แต่เมื่อขอมาก็จัดให้ครับ
1. จงพิสูจน์ว่า $\frac{x^4}{1+x^8} \leq \frac{1}{2} $
2. จงพิสูจน์ว่า $\sqrt{\frac{x^2+y^2}{2}}\geq \frac{x+y}{2}$
3. สำหรับจำนวนนับ n ใดๆ จงแสดงว่า $(n+1)^n\geq 2^n n!$
ลองดูสัก 3 ข้อก่อนก็แล้วกันครับ

CmKaN 12 ตุลาคม 2007 20:34

ยังยากอยู่ดีครับ:confused: ค่อนข้างมึน ว่าจะจับคู่อะไรออกมาดี ท่านหยินยางมีเทคนิคอะไรไหมครับ

1. $ \frac{x^{8}+1}{2} \geq \sqrt{(x^{8})(1)}$
$\frac{x^{4}}{x^{8}+1} \leq \frac{1}{2} $
อีกสองข้อยังคิดไม่ออกเลยครับ:confused:

หยินหยาง 12 ตุลาคม 2007 21:07

อ้างอิง:

ข้อความเดิมเขียนโดยคุณ CmKaN (ข้อความที่ 23487)
ยังยากอยู่ดีครับ:confused: ค่อนข้างมึน ว่าจะจับคู่อะไรออกมาดี ท่านหยินยางมีเทคนิคอะไรไหมครับ

1. $ \frac{x^{8}+1}{2} \geq \sqrt{(x^{8})(1)}$
$\frac{x^{4}}{x^{8}+1} \leq \frac{1}{2} $
อีกสองข้อยังคิดไม่ออกเลยครับ:confused:

hint
ข้อ 2 ใชัแนวคิดของ A.M.-G.M. โดย $x^2+y^2 \geq 2xy$
ข้อ 3. ใชัแนวคิดของ A.M.-G.M. โดย $\frac{1+2+3+...+n}{n} \geq\sqrt[n]{1*2*3...*n} $
ขอให้สนุกกับการทำโจทย์ครับ

nooonuii 12 ตุลาคม 2007 23:44

การใช้อสมการ $AM-GM$ จะขึ้นอยู่กับจำนวนตัวแปรครับ เวลาพิสูจน์อสมการให้มองที่ตัวเลขที่ปรากฎอยู่ในอสมการครับ

ข้อ 1 ในอสมการมีตัวเลข $2$ ปรากฎอยู่ แถมกำลังของตัวแปรตัวหนึ่งก็เป็นสองเท่าของอีกตัวหนึ่ง จึงควรคิดว่าน่าจะใช้ AM-GM สำหรับ $2$ ตัวแปร จากนั้นเราก็พยายามจัดรูปอสมการให้เข้ากับ $AM-GM$ สำหรับ $2$ ตัวแปรให้มากที่สุด แล้วเราจะเห็นว่าควรจะเลือกตัวแปรสองตัวนั้นเป็นอะไรครับ

ข้อ 2 ในอสมการมีตัวเลข $2$ ปรากฎอยู่หลายที่ แถมยังมีรากที่ 2 ด้วยอีกต่างหากซึ่งใกล้เคียงกับรูปแบบของอสมการ AM-GM สำหรับ 2 ตัวแปรมากๆ อย่างนี้ก็ควรคิดว่าน่าจะใช้ AM-GM สำหรับ $2$ ตัวแปร แต่อาจจะต้องมีการจัดรูปอสมการอีกเล็กน้อย ซึ่งส่วนใหญ่ก็ใช้วิธีทำย้อนกลับครับ

ข้อ 3 ตัวแปรที่โดดเด่นที่สุดในอสมการคือ $n$ ครับ อย่างนี้คงต้องใช้ .........??
วิธีการนี้อาจจะใช้ไม่ได้เสมอไปครับ บางครั้งเราก็ต้องดูองค์ประกอบอย่างอื่นด้วยเช่นอสมการ
$$\frac{x}{2}+\frac{2}{x^2}\geq\frac{3}{2}$$
มี $2$ อยู่เต็มไปหมด แต่ต้องใช้ AM-GM $3$ ตัวแปร ซึ่งก็เป็นไปได้เพราะ $3$ ก็ปรากฎอยู่ในอสมการ บางครั้งอสมการมีการยุบรวมเทอมที่เหมือนกันมาไว้ด้วยกันก็อาจจะทำให้มองยากขึ้นครับ อสมการนี้ถ้าผมเขียนว่า
$$\frac{x}{4}+\frac{x}{4}+\frac{2}{x^2}\geq\frac{3}{2}$$
ก็น่าจะทำได้ใช่ไหมครับ

คงต้องฝึกทำโจทย์เยอะๆครับ แล้วประสบการณ์จะช่วยให้เราเข้าใจแนวคิดมากขึ้น แถมโจทย์ให้ครับ
ให้ $a,b,c>0$ พิสูจน์ว่า
4. $a^3+3b^3+9c^3\geq 9abc$

5. $\dfrac{1}{1+a}+\dfrac{1}{1+b}+\dfrac{1}{1+c}\geq\dfrac{9}{3+a+b+c}$

6. $(1+a^2)(1+b^2)(1+c^2)\geq 8abc$

CmKaN 15 ตุลาคม 2007 07:53

ขอบคุณ พี่noonuii กับท่านหยินหยางมากครับ:)
คิดออกข้อ6.จับ $1+a^{2},1+b^{2},1+c^{2}$แล้วบวกกัน
ข้อ4,5เหมือนเกิอบออกอ่ะครับแต่ยังไม่ออกอ่ะครับ เดี๋ยวไปลองคิดดูใหม่ล่ะกัน

ปล.ขอโจทย์เพิ่มได้ไหมครับ เพื่อคิดออกข้ออื่นๆจะได้มีแนวทางมากขึ้น

CmKaN 15 ตุลาคม 2007 18:12

พิมพ์ผิดครับ:please: ขอโทษทีครับ:kiki:
5.$ \frac{1}{1+a}+\frac{1}{1+b}+\frac{1}{1+c} \geq \frac{9}{3+a+b+c}$

(AM-GM) $\frac{(1+a)+(1+b)+(1+c)}{3} \geq \sqrt[3]{(1+a)(1+b)(1+c)} ---(1)$
(AM-GM) $\frac{\frac{1}{1+a}+\frac{1}{1+b}+\frac{1}{1+c}}{3} \geq \sqrt[3]{({\frac{1}{1+a})(\frac{1}{1+b})(\frac{1}{1+c})}} ---(2)$
$(1) \times{(2)} \frac{[\frac{1}{1+a}+\frac{1}{1+b}+\frac{1}{1+c}][(1+a)+(1+b)+(1+c)]}{9} \geq \sqrt[3]{\frac{(a+1)(b+1)(c+1)}{(a+1)(b+1)(c+1)}}=1$
$ \frac{1}{1+a}+\frac{1}{1+b}+\frac{1}{1+c} \geq \frac{9}{3+a+b+c}$

CmKaN 15 ตุลาคม 2007 20:24

คิดข้อ4ออกแล้วครับ
4.$a^{3}+3b^{3}+9c^{3} \geq 9abc$
$\frac{a^{3}}{3}+\frac{a^{3}}{3}+\frac{a^{3}}{3}+3b^{3}+9c^{3} \geq 9abc$
(AM-GM) $\frac{a^{3}}{3}+b^{3}+3c^{3} \geq 3 \sqrt[3]{a^{3}b^{3}c^{3}} = 3abc$
ในทำนองเดียวกันจะได้เหมือนกัน 3 อสมการ จากนั้นนำมากบวกกันจะได้$a^{3}+3b^{3}+9c^{3} \geq 9abc$

nooonuii 15 ตุลาคม 2007 21:30

อ้างอิง:

ข้อความเดิมเขียนโดยคุณ CmKaN (ข้อความที่ 23557)
คิดข้อ5ออกแล้วครับ
5.$ \frac{1}{1+a}+\frac{1}{1+b}+\frac{1}{1+c} \geq \frac{9}{3+a+b+c}$

(AM-GM) $\frac{(1+a)(1+b)(1+c)}{3} \geq \sqrt[3]{(1+a)(1+b)(1+c)} ---(1)$
(AM-GM) $\frac{\frac{1}{1+a}+\frac{1}{1+b}+\frac{1}{1+c}}{3} \geq \sqrt[3]{\frac{1}{({\frac{1}{1+a})(\frac{1}{1+b})(\frac{1}{1+c})}}} ---(2)$
$(1) \times{(2)} \frac{1}{1+a}+\frac{1}{1+b}+\frac{1}{1+c} \geq \frac{9}{3+a+b+c}$

ไม่เข้าใจครับ :confused:

nongtum 15 ตุลาคม 2007 22:26

ข้อ 5 ของคุณ CmKaN ตรง AM-GM เทอมทางซ้ายมือต้องบวกกัน ไม่ใช่คูณกัน
อีกอย่างพอจับคูณกันบรรทัดสุดท้าย มันไม่ได้สมการโจทย์นะครับ

CmKaN 16 ตุลาคม 2007 07:48

แก้ไขแล้วครับ:)

nooonuii 16 ตุลาคม 2007 09:43

ข้อ 4 ยังทำยาวเกินไปนะครับ ใช้ AM-GM แค่ครั้งเดียวก็ออกแล้วครับ

CmKaN 16 ตุลาคม 2007 18:09

อืมคูณสามได้เลยใช่ไหมครับ ถ้ามีเวลาขอโจทย์อีกได้ไหมครับ หรือไม่ก็แหล่งที่มีโจทย์ก็ได้ครับ:please: ขอบคุณครับ

หยินหยาง 16 ตุลาคม 2007 21:30

อ้างอิง:

ข้อความเดิมเขียนโดยคุณ CmKaN (ข้อความที่ 23583)
อืมคูณสามได้เลยใช่ไหมครับ ถ้ามีเวลาขอโจทย์อีกได้ไหมครับ หรือไม่ก็แหล่งที่มีโจทย์ก็ได้ครับ:please: ขอบคุณครับ

จัดให้อีกรอบครับ
7. กำหนดให้ $x, y, z $เป็นจำนวนจริงจงแสดงว่า $x^4+y^4+z^4\geq xyz(x+y+z)$
8. กำหนดให้ $a, b, c $เป็นจำนวนจริงบวกจงแสดงว่า $a+b+c\leq \frac{bc}{a}+\frac{ca}{b}+\frac{ab}{c}$
9. กำหนดให้ $a, b, c $เป็นจำนวนจริงบวก จงแสดงว่า $a(1+b)+b(1+c)+c(1+a) \geq 6\sqrt{abc} $

CmKaN 18 ตุลาคม 2007 19:35

8. $a+b+c\leq \frac{bc}{a}+\frac{ca}{b}+\frac{ab}{c}$
หาร $abc$ ตลอด $\frac{1}{a^{2}}+\frac{1}{b^{2}}+\frac{1}{c^{2}} \geq \frac{1}{bc}+\frac{1}{ac}+\frac{1}{ab}$
AM-GM $\frac{(\frac{1}{a^{2}})+(\frac{1}{b^{2}})}{2} \geq \sqrt{\frac{1}{a^{2}b^{2}}} = \frac{1}{ab}$
ในทำนองเดียวกันจะได้
$\frac{(\frac{1}{a^{2}})+(\frac{1}{c^{2}})}{2} \geq \frac{1}{ac},$
$\frac{(\frac{1}{b^{2}})+(\frac{1}{c^{2}})}{2} \geq \frac{1}{bc}$
นำอสมการทั้งสามมาบวกกันจะได้$2(\frac{1}{a^{2}}+\frac{1}{b^{2}}+\frac{1}{c^{2}}) \geq 2(\frac{1}{bc}+\frac{1}{ac}+\frac{1}{ab})$
$\therefore a+b+c\leq \frac{bc}{a}+\frac{ca}{b}+\frac{ab}{c}$

9. $a(1+b)+b(1+c)+c(1+a) \geq 6\sqrt{abc}$
AM-GM $\frac{(a)+(bc)}{2} \geq \sqrt{abc}--(1)$
AM-GM $\frac{(b)+(ac)}{2} \geq \sqrt{abc}--(2)$
AM-GM $\frac{(c)+(ab)}{2} \geq \sqrt{abc}--(3)$
$(1)+(2)+(3) a+ab+b+bc+c+ac \geq 6\sqrt{abc}$
$\therefore a(1+b)+b(1+c)+c(1+a) \geq 6\sqrt{abc}$


เวลาที่แสดงทั้งหมด เป็นเวลาที่ประเทศไทย (GMT +7) ขณะนี้เป็นเวลา 02:29

Powered by vBulletin® Copyright ©2000 - 2024, Jelsoft Enterprises Ltd.
Modified by Jetsada Karnpracha